You are on page 1of 20

Investment Planning Mock Test: Practical Questions

Section 1: 41 Questions (1 Mark each):

1) (A) Beta reflects the systematic risk of the portfolio.


(B) Beta reflects the unsystematic risk of the portfolio. .
A) (A) is correct.
B) (B) is correct.
C) Both (A) and (B) are incorrect.
D) Both (A) and (B) are correct.

2) For maintaining flexible ratio of asset allocation, one should ________________.


A) rebalance the debt/equity allocation very frequently
B) keep fixed percentage of equity and debt investments at all times
C) generally avoid portfolio rebalancing
D) rebalance the debt/equity allocation periodically

3) (A) Constant proportion portfolio insurance policy (CPPI policy) is the worst policy if
the stock market moves in only one direction, either up or down.
(B) Constant proportion portfolio insurance policy (CPPI policy) is the best policy if the
stock market moves in only one direction, either up or down. .
A) (A) is correct.
B) (B) is correct.
C) Both (A) and (B) are correct.
D) Both (A) and (B) are incorrect.

4) Principal vectors of an active portfolio strategy are:


A) Market timing.
B) Sector rotation.
C) Security selection.
D) All of the above.

5) What is the reliable source of pertinent information for the investor in Mutual fund
scheme?
A) Advice of distributor.
B) AMFI website.
C) Financial press.
D) Offer Document.

Financial Planning Academy


6) (A) HPR (Holding Period Return) = (Cash payments received during the period +
Change in price) / Beginning Price.
(B) HPR = (Cash payments received during the period + price at the end of the period /
Beginning Price.
A) (A) is correct.
B) (B) is correct.
C) Both (A) and (B) are correct.
D) Both (A) and (B) are incorrect.

7) (A) For a Call option: Intrinsic Value = Spot price - Strike Price. Intrinsic value must
be positive or zero.
(B) For a Put Option: Intrinsic Value = Strike price - Spot Price. Intrinsic value must be
positive or zero.
A) (A) is correct.
B) (B) is correct.
C) Both (A) and (B) are correct.
D) Both (A) and (B) are incorrect.

8) The modern portfolio theory suggests that the portfolio returns can be optimized
by _________.
A Laddering the bond portfolio.
B Investing in diversified equity funds.
C Moving closer to the efficient frontier in terms of the risk return equation.
D Investing in treasury bills and equities.

9) If one is a fundamental analyst, he/she would follow which one of the following
approaches?
A Passive diversified portfolio approach
B Quantitative Analysis (searching for undervalued securities)
C Efficient market hypothesis-strong
D Efficient market hypothesis-weak

10) Derivatives on gold related securities and government securities that are traded on
the stock exchanges are regulated by _______.
A SEBI
B RBI
C Department of Company Affairs
D Company Law Board

Financial Planning Academy


11) The bid-ask spread is________.
1) Broker's commission
2) Dealer's gross income from a transaction
3) Larger for illiquid securities than for liquid ones.
A1
B2
C3
D 1, 2 & 3.

12) Priti wants to invest in XYZ Ltd's stock, however, she must wait several months
till her Fixed Deposit matures. What type of option should she invest in to protect
against the market value of the stock increasing before her money becomes available?
A Buy a call option
B Buy a put option
C Write a put option
D Sell a call option

13) The risk of the writer of an option is _______.


A Market price at expiry
B Purchase price
C Limited by the premium
D Unlimited

14) MFs are allowed to participate in securities lending.


A False
B True
C Sometimes
D Cannot say

15) The companies in which significant amount of shares are held by the parent who
also exerts some influence in the activities is known as__________.
A Associated companies
B Wholly owned subsidiaries
C Partly owned subsidiaries
D Unlimited companies

16) Reciprocal of P/E ratio, E/P ratio measures___________


A Market price of a share
B Growth ratio
C Cost of debt
D Opportunity Cost of capital

Financial Planning Academy


17) For the Previous Year 2006-07, XYZ Ltd., a domestic company, pays an interim
dividend in October 2006 of 15% and a final dividend of 40%. The distribution tax is
payable on _________.
A Both interim and final dividend
B Only the final dividend
C Only the interim dividend
D On neither the interim nor the final dividend

18) Interest rate on Post Office Savings Bank is payable__________.


A) monthly
B) quarterly
C) half yearly
D) Yearly

19) A convertible preference share, depending upon the terms of issue, may be converted
into a common share at the option of the shareholder_________.
(i) at a conversion ratio that never changes
(ii) within a limited number of years after the preferred stock is issued
(iii) only after a specified number of years have elapsed since the preference share was
initially issued
A) (i) only
B) (i), (ii) & (iii)
C) (i) & (ii) only
D) (i) & (iii) only

20) (i) Income received by Mutual Fund is Tax free.


(ii) Dividend income received by investors from Mutual Funds is Tax free.
A) Only (i) is correct.
B) Only (ii) is correct.
C) Both (i) & (ii) are incorrect.
D) Both (i) & (ii) are correct.

21) An Option premium is_______.


(i) market price of an underlying asset
(ii) difference between the current share price and the strike price
(iii) profit made on the Option
A) Only (iii) is true.
B) Only (ii) is true.
C) Both (i) & (ii) are true.
D) None of the above.

Financial Planning Academy


22) A proprietary company can be a subsidiary of a listed company.
A) False.
B) True.
C) Cannot say.
D) Sometimes.

23) Among the four bonds given below, each having a 20 years maturity, which one is
the most volatile? .
A) Municipal.
B) Treasury.
C) Zero Coupon.
D) BB-rated corporate.

24) Senior Citizens Savings Scheme is governed by which Act?


A) Banking Regulation Act, 1949
B) Government Savings Bank Act, 1873
C) Small Savings & Misc. Savings Act, 1902
D) Companies Act, 1956

25) If the current share price is S and the set exercise price is X, the intrinsic value of the
Call Option is ______.
A) Max ( 0, S-X ).
B) Max ( 0, X-S ).
C) Min ( 0, S-X ).
D) Min ( 0, X-S ).

26) A PPF account was opened on 10th July 1990. It will mature on ________.
A) 11/07/2005
B) 10/07/2005
C) 09/07/2005
D) 01/04/2006

27) Convertible debentures are valued on the basis of Market Value.


A) This statement is false.
B) This statement is true.
C) Cannot say.
D) Sometimes.

28) T-bills pay interest to their investors by __________.


A) T-bills pay no interest.
B) Coupon interest.
C) Possible price appreciation above their discounted price.
D) Difference between Issue price and face value.

Financial Planning Academy


29) Akash is a self employed, moderate risk taking person. He is 10 years away from
retirement. Before he decides to invest he has come to you to educate himself regarding
investment risk, return and diversification. Akash wants to know which among the
following investment avenues normally has highest number of compounding periods in a
year.
A PPF
B Time Deposit
C NSC
D RBI Bonds

30) All stock market indexes are most accurately characterized by which of the following
statements about the degree to which they co-vary together?
A They are uncorrelated.
B They are perfectly positively correlated.
C They are highly positively correlated.
D They are negatively correlated.

31) Any lead manager for an IPO of a company requires_______.


A SEBI Category-2 Certificate with 2% fee of issue amount
B SEBI Category-1 Certificate
C SEBI Category-2 Certificate
D SEBI Category-3 Certificate with 2.5% fee of issue amount

32) An equity shareholder has the following right_______.


1) He can legally demand information from the company and gain access to its books.
2) He can vote for the common shareholders. dividend.
3) He can vote for the preference shareholders. dividend.
A1
B2
C3
D 1, 2 & 3

33) A Preference Shareholder enjoys priority over common stockholders in respect


of_______.
1) Dividends cannot be paid to common stockholders unless he receives the stated
dividend.
2) In the event of liquidation, he is paid before the common shareholders.
3) He can elect the Chairman of the Board of Directors.
4) He can vote for dividend to be paid to the equity shareholders.
A Both (3) & (4) are true, but (1) & (2) are false.
B (1), (2), (3) & (4) are true.
C Both (1) and (2) are true, but (3) and (4) are false.
D None of the above

Financial Planning Academy


34) In the derivatives market, hedgers, speculators and arbitrageurs trade. Hedger is a
trader _______.
A who enters derivatives market in order to reduce a pre-existing risk
B who enters derivatives market in anticipation of a need in the near future
C whose net wealth change, at the time the derivative contract expires, is expected to be
zero (Perfect hedge assumed)
D All of the above

35) Which of the statements given below is not correct with regard to futures
transactions?
A Cash flow management.
B Leveraging exposure.
C Benefit from premium payments.
D Portfolio substitution.

36) If an investment has a correlation coefficient of 0.80 with the market, which of the
following performance measures is the best measure of risks?
A) Jensen.
B) Sharpe & Jensen.
C) Sharpe.
D) Treynor.

37) Bond prices are sensitive to the coupon rate of the bond and the maturity term of the
bond. Bond prices are less sensitive to changes in interest rates when the bonds
have_______.
A) large coupons and short maturity
B) large coupons and long maturity
C) small coupons and short maturity
D) small coupons and long maturity

38) In the calculation of the present value of shares, increasing the growth factor (all
other things being equal) will cause the value to________.
A) Stay the same
B) Decrease
C) Increase
D) Increase then Decrease

39) If a new issue was offered to the public at 15 times earnings but the market was
pricing similar shares at 19 times, this would be_______.
A) an example of low gearing
B) a situation where the investor cannot take a position
C) an appalling proposition to the investor
D) a bargain not to be missed

Financial Planning Academy


40) Under the efficient market hypothesis, which of the following term best describes the
movement of stock prices?
A) Statistical.
B) Random.
C) Diverse.
D) Predictable.

41) A share holder who has paid for the first call, in a limited company, is liable for
_______.
A) his share in the total liabilities of the company
B) partially paid-up amount of the shares allotted to him
C) fully paid-up value of the shares allotted to him
D) only the secured debt of the company

Financial Planning Academy


Section 2: 30 Questions (2 Mark each):

1) A 28 year old bachelor earning well for his age goes to a CFPCM Certificant for
financial advice. He tells the Financial Planner that he does not want to look beyond 2
years and all he wants is advise on how to maximize his returns. The most appropriate
action is likely to be:
(A) Help him to invest in equity markets.
(B) Assist him in investing in debt instruments of appropriate maturity.
(C) Explain to him the risks of equity and debt markets. .
A) Only (C)
B) Only (B)
C) Both (A) & (C)
D) Both (B) & (C)

2) Security (A) has 12% return and 7% Standard Deviation (SD), Security (B) has 18%
return and 8% SD A conservative investor should prefer _______________________.
A) Security (A) since it has 95% chance of giving +ve return.
B) Security (B) since it has 95% chance of giving +ve return.
C) Security (A) has lower Standard Deviation.
D) Security (A) has lower Variance of return.

Solution: It is based on Normal Distribution theory, ie Bell Theory, given in P. Chandra


book.)
Expected Return = Return on security - 2* SD
Expected Return of Sec. A. = 10 - 2*6 => Sec. A = - 2 %
Expected Return of Sec. B = 18 - 2*8 => Sec B = 2 %
Therefore there is 95% chance Sec. B will get Positive Return.

3) Find Beta of security X if expected market premium is 15%, risk free return is 7% and
expected return of security X is 20%? .
A) 0.834
B) 0.900
C) 0.700
D) 0.867

Solution: Rf = 7, Market Risk Premium = (R m – Rf), Rx = 20


Rx = Rf + (R m – Rf) * β
20 = 7 + 15 * β
β = 0.867

Financial Planning Academy


4) Shanu buys a call option of Infosys 2040 at Rs.35 and sells a call option of 2100 at
Rs.15. The lot size of Infosys is 100. What is the Maximum profit for Shanu? Ignore
brokerage, STT, Service tax etc.
A) Rs. 4000
B) Rs. 2000
C) Rs. 2500
D) Rs. 6000

Solution: Strike price of purchased call option 2040


Strike price of sold call option 2100
Difference 60

Initial Cash flow:


Bought option -35
Sold option +15
Difference -20
Maximum profit 60-20=40 per unit i.e 4000.

5) A companys current dividend is Rs. 6. However it is constantly falling @ 5% per


annum. If the discount rate is 15% what is the value of its share?
A) Rs. 63.00
B) Rs. 23.75
C) Rs. 28.50
D) None of the above.

Solution: Use Gordon Constant Growth Model


P.V = D1/(r – g) where D1 = D0(1 + g) = 6 * (1-0.05) = 5.7, r = 15, g = - 5
P.V = 28.5

6) If the post tax rate of return on an investment is 8% and the inflation rate is 5%
the real rate of return is _______.
A 3.5%.
B 3%.
C 2.86%.
D -3%

7) A 6% 2014 GOI Bond of Rs.100 Face Value trades at Rs.102 on 15/12/2004. There is
a Call Option in 2008. The Yield to Call will be ________ than the Yield to Maturity.
A Higher
B Lower
C Same
D Cannot Say

Financial Planning Academy


8) A treasury bill of face value of Rs. 1,00,000/- is selling at Rs. 97,500/- today. It is
slated to mature in 60 days. The annual yield is _______.
A 13.3%
B 15.59%
C 16.43%
D 14.27%

Solution: Annual Yield = (Future Value/Present Value - 1) * (365/N)


Annual Yield = (100000/97500 – 1) * (365/60) = 15.59%

9) If the Buying price of a property is Rs. 20 lakh, Net income is Rs. 2 lakh and
Market yield is 8%, the value of the property is Rs. __________.
A 20 lakh
B 22 lakh
C 25 lakh
D None of the above

Solution = Present Value of Property = Net Income / Market Yield = 2 / 0.08 = 25 lacs

10) A company offers a rights issue of one for two for Rs. 7 each. The current market
price is Rs. 13. The expected ex-right market price would be Rs. _______.
A 9
B 10
C 11
D None of the above

Solution: Rights issue: For every 2 shares we get 1 share.


Current Market Price for 2 shares = 2 * 13 = 26, Price for rights share = 7
Total price for 2 shares and rights share = 33
Ex-right Market price = 33/3 = 11

11) An MF scheme can borrow money up to ___% of its Asset Under Management
for maximum ___ months.
A 30%, 3 months
B 30%, 12 months
C 10%, for any period
D 20%, 6months

12) Assume a company has issued Ten lakh equity shares and its current market price is
Rs. 80. Last years. profit was Rs. 90 lakh out of which Rs. 50 lakh was distributed as
dividend. What is the earning yield?
A) 8.89%
B) 11.25%
C) 6.25%
D) 16.67%

Financial Planning Academy


Solution: Profit per share = 90 lacs / 10 lacs = 9, Current Market Price = 80
Earning Yield = Profit per share / Current Market Price = 9 / 80 = 11.25 %

13) While making an investment in a Debt fund for your client you make the following
remarks :
(i) Rate of return by the instrument in past is 8.5% and is expected to continue in future.
(ii) Investment would have low risk.
Which of the statement is voilative with respect to CFP Code of Ethics?
A) (i) Only
B) (ii) Only
C) Both (i) and (ii)
D) None of the above

14) Mohan invested a sum for 5 years @ 8% p.a where it was compounded annually for
the first 4 years and quarterly for the last 1 year. He received Rs. 85,000 on maturity.
What was the amount he had initially invested?
A) Rs. 4,16,000
B) Rs. 78,527
C) Rs. 57,720
D) Rs. 57,203

15) Have a look at the data given below and answer the query appearing below .
Probability Security A (Returns in %) Security B (Returns in %)
0.1 30 40
0.25 20 25
0.3 0 15
0.25 -5 0
0.1 -10 -20
Standard Deviation of the Portfolio for equal weightage of securities A & B is________
A) 10.29
B) 11.69
C) 12.88
D) 14.30

16) If Reserve Bank of India wants to lower the interest rates in the economy, it would
consider which of the following option/s?
(A) Purchase government securities.
(B) Increase the reserve requirement of member banks.
(C) Increase the discount rate.
(D) Decrease the reserve requirement of member banks.
A) (A) & (B) only
B) (A) & (D) only
C) (B) & (C) only
D) (A) only

Financial Planning Academy


17) Consider a portfolio of two investments viz. A & B. The sum total of volatility of A
and B respectively, represented by standard deviation of the two investments, will be
equal to the volatility of the portfolio as a whole if _________.
A) A and B have a correlation of 1
B) the portfolio is equally divided between A and B
C) A and B have a correlation of Zero
D) the return on the portfolio is equal to the sum of returns of A and B

18) Risk free rate of return is 8%, expected market premium is 15% and Beta of security
is 0.80. What is the expected rate of return of the security? .
A) 13.6%
B) 15.00%
C) 12.00%
D) 20.00%

Solution: Market Risk Premium = (R m – Rf) = 15 , β = 0.8, Rf = 8


Rx = Rf + (R m – Rf) * β = 8 + 15 * 0.8 = 20 %

19) If a Corporation declares and pays dividend, this transaction will________.


A) reduce stockholders equity
B) increase liabilities
C) decrease net income
D) not affect total assets

20) Which of the following technical indicators measure the strength of the market by
comparing the number of stocks that advance or decline in a particular trading day? .
A) Support level.
B) Breadth of the market.
C) Short interest.
D) Call-Put ratio.

21) Dividend received by the writer of a Call during the life span of an Option
__________.
A) can be pocketed by the writer
B) has to be given to the buyer of the option, only if he eventually exercises the option
C) increases the premium value by an equivalent amount
D) decreases the strike price

22) Mr. A owes Rs.50000 to you and the interest is 12% pa compounded monthly. What
would you prefer?
A Take Rs. 60000 after 1 year
B Take Rs. 25000 now and Rs. 32000 at the end of the year.
C Take Rs. 40000 now and Rs.15,200 at the end of the year.
D Take Rs. 50000 now

Financial Planning Academy


23) You buy a growth-oriented non-dividend paying share for Rs. 200 and 4 years later
you sell it for Rs. 350. The compound annual growth rate is _____.
A 10.3%
B 18.8%
C 75%
D 15%

24) Manish estimates his opportunity cost on investments to be 12% compounded


annually. Which one of the following is the best investment opportunity for Manish?
A To receive Rs. 5000 at the beginning of each six-month period for nine years
compounded semiannually.
B To receive Rs. 40000 at the end of four years and Rs. 120000 eight years later (at the
end of the 12th year).
C To receive Rs 50000 today.
D To receive Rs. 250000 at the end of 14 years.

25) Mr. Hitesh Shah has a portfolio with 23 different equities. The portfolio increased by
20% and has a beta of 1.50. Utilizing the Capital Asset Pricing Model, compute by what
percent the market changed (round to nearest 0.5%) (Assume risk free rate of 5%)?
A 14%
B 15%
C 16.5%
D 17%

Solution: Market Risk Premium = (R m – Rf), β = 0.8, Rf = 5, Rp = 20


Rp = Rf + (R m – Rf) * β
20 = 5 + (R m – 5) * 1.5
R m = 15

26) The August call option exercise price of HLL is Rs. 350. The current share price is
Rs. 360. The premium is Rs. 15. The intrinsic value of the call, ignoring transaction costs,
is Rs. _______.
A Nil
B5
C 10
D 15

27) Zarina invested Rs. 45000 in stocks at the beginning of the year. She received bonus
shares worth Rs. 4000 at the year end. The value of the portfolio at the end of the year
was Rs. 47000. Calculate the returns of the portfolio during the year?
1 8%
2 - 4.45%
3 5.5%
4 - 7.2%

Financial Planning Academy


28) Risk free return of Security A is 8%. You expect that the return on market would be
14%. The expected return of Security A with Beta of 0.70 is ________.
A) 17.8%
B) 15.4%
C) 12.2%
D) 18.2%

Solution: Rx = Rf + (R m – Rf) * β = 8 + (14 – 8) * 0.7 = 12.2 %


29) Consider that an investor writes a covered call on XYZ share. Spot price is Rs. 38,
Exercise price is Rs. 40 and a 3 month call on XYZ share is traded at Rs. 3. What is the
initial cash flow incurred at the time of investment?
A) Rs. 37
B) Rs. 41
C) Rs. 38
D) Rs. 35

30) The NAV of a debt oriented Mutual Fund is 22.25 cum dividend. It has declared a
dividend of 6%, record date being 25th June 2006. Calculate the dividend receivable by
an individual investor if he holds 10,000 units.
A) Rs. 4,653.60
B) Rs. 6,000.00
C) Rs. 5,158.50
D) Rs. 5,250.00

Solution = NAV = 22.25, Dividend per unit = 6% of Face Value = 0.06 * 10 = 0.6
Dividend for 10,000 units = 6000,
Dividend receivable by investor = Dividend for 10,000 units – DDT(for debt oriented
mutual fund = 14.025%) = 6000 – 0.14025 * 6000 = 6000 – 841.5 = 5158.5

Financial Planning Academy


Section 3: 16 Questions (4 Mark each):

1) Sunil has purchased 100 convertible debentures of XYZ on 1/1/2001 at Rs. 500 each.
30% of the value of debentures in convertible into one share of Rs. 50 each after 6 years.
Sunil exercised his option on 1/1/2007 and received 100 shares. Compute the cost of
acquisition off these shares.
A) Rs. 150
B) Rs. 500
C) Rs. 155
D) Rs. 145

Solution: 30% of value of debentures = 0.3 * 100 * 500 = 15000


No. of shares received on exercise of option = 100
Cost of Acquisition of Shares = 15000 / 100 = 150

2) A Firm's Current Ratio is 1.4, Current Liability is Rs. 1600, Acid Test Ratio is 1.2, and
Inventory Turnover Ratio is 8. What would be the cost of goods sold for the firm? .
A) Rs. 2240
B) Rs. 1920
C) Rs. 2560
D) Rs. 2750

Solution: Current Ratio = Current Assets / Current Liabilities


Current Assets = 2240
Acid Test Ratio = Quick Assets / Current Liabilities
Quick Assets = 1920 = Current Assets – Inventories
Inventories = 320
Inventory Turnover Ratio = Cost of goods sold / Inventories
Cost of goods sold = 8 * 320 = 2560

3) Balvinder placed Rs. 10000 with a mutual fund which charged him a load of 2.25%.
Management and other fees charged is 1.10%. Ignoring other costs over 5 years, what
annual returns would the fund have to produce to equal the value that the investment
would have earned in 8% fixed deposit?
A) 8.59%
B) 9.00%
C) 8.00%
D) 9.59%

Solution: Step 1) Value of Investment with 8% FD:


PV = 10000, N = 5, I = 8,FV = 14693.28
Step 2) For this Accumulated amount FV = 14693.28, N = 5,
PV = -10000* (1-2.25%) = 9775, I(Return) = 8.49%
Annual return = 8.49% + 1.1%(Annual charges) = 9.49%

Financial Planning Academy


4) If the net present value of a series of discounted cash flows is less than zero, one
could interpret that:
1. The discounted cash flows are lower than the investment outlay
2. The rate of return is lower than the cost of capital
3. The return of investment is higher than the internal rate of return
4. The internal rate of return was the discount rate used 4
A 1, 2, 3 & 4
B 2 and 3 only
C 1 and 4 only
D 1 only

5) Security A has a standard deviation of 23% and the market has a standard deviation of
18%. The correlation coefficient r between Security A and the market is 0.80. What is the
% of the change in Security A can be explained by changes in the market?
A 80%
B 50%
C 36%
D 64%

Solution = % of change in Security A explained by change in market = Square of


correlation coefficient = 0.8 * 0.8 = 64 %

6) Which of the following statements is/are true regarding strategic and tactical asset
allocation?
1) Strategic asset allocation involves selection of the correct asset allocation based on risk
tolerance of the client, economic forecasts, and expectations of selected asset classes and
rebalancing once or twice per year to keep the portfolio within the parameters of the
desired strategic mix.
2) Tactical asset allocation involves evaluating asset classes or industries as to their value
and selling undervalued classes and purchasing overvalued classes.
A 1 only is true
B 2 only is true
C Both 1 & 2 are true
D Neither 1 nor 2 is true

Financial Planning Academy


7) You are evaluating the following table for your customer:
Fund Average Returns (%) Standard Deviation (%) Beta
A 22 24 1.3
B 14 18 0.9
C 16 17 1.1
Market 12 15 1
Assume risk-free rate is 6%.The rankings based on the Treynor Ratio (in the order
of best to worst) is ___________.
A A, B, C
B A, C, B
C B, A, C
D C, B, A

8) Returns on a security held for 5 years by Praveen are: First year 7%, second year 3%,
third year -9%, fourth year 6%, fifth year 10%. Find the standard deviation of the
security.
A) 8.50%
B) 7.37%
C) 8.00%
D) 6.59%

9) The XYZ Company paid a dividend of Rs. 2 last week. The dividend for XYZ
company is expected to be 100% greater next year. The growth rate for the following
year is expected to be 50%. In the third year and thereafter, the dividend is expected to
grow annually at a rate of 8%. If your requirement is a retu
rn of 12%, what is the maximum you would pay for the security?
A) Rs. 159.50
B) Rs. 142.70
C) Rs. 137.50
D) Rs. 140.50

10) Shashi wants to purchase a car which is costing Rs. 8,50,000. Reviewing her budget
she determines she can afford to pay Rs. 15,000 per month for three years towards the
car. The going rate of interest is 1% per month for 3 years. How much can she afford to
borrow? Indicate the nearest amount.
A) Rs. 4,50,000
B) Rs. 5,50,000
C) Rs. 8,00,000
D) Rs. 6,00,000

Solution: PMT = -15000, N = 3 * 12, I = 1%, PV = 4,51612.57 (approx 450000)

Financial Planning Academy


11) The May futures contract on XYZ Ltd closed at Rs. 3,940 yesterday. It closes today
at Rs. 3,898.60. The spot closes at Rs. 3,800. Raju has a short position of 3,000 in the
May futures contract. He sells 2,000 units of May expiring put options on XYZ with a
strike price of Rs. 3,900 for a premium of Rs. 110 per unit. What is his net obligation
to/from the clearing corporation today? .
A) Pay Rs. 3,44,200.
B) Receive Rs. 3,44,200.
C) Receive Rs. 6,40,000.
D) Pay Rs. 95,800.

Solution: Closing price yesterday = 3940


Closing price today = 3898.6
Price Difference = 41.4
Contract size = 3000
Net Obligation from corporation = 3000 * 41.4 = 124200
Premium for selling = 110 per unit, Units sold = 2000
Proceeds from sale = 2000 * 110 = 220000
Net obligation from corporation = 124200 + 220000 = 344200

12) ABC Ltd.’s stock has a current dividend of Rs. 1.75 that is growing at 8%. If the
stock is currently selling for Rs. 100 and your required rate of return is 10%, will you buy
the stock at today’s price?
A Yes, because the stock is a good buy based on a risk-return relationship.
B No, because the stock is overvalued based on the dividend growth model.
C No, because the stock is a bad investment based on a risk-return relationship.
D Yes, because the stock is undervalued based on the dividend growth model.

13) A portfolio consists of 3 securities 1, 2 & 3. The proportion of those securities are
w1=0.3, w2=0.5 & w3=0.2. The Standard Deviation of those securities are SD1 = 6%,
SD2 = 9% and SD3= 10%. The correlation coefficient among these securities are r12 =
0.4, r13 = 0.6 and r23 = 0.7. Calculate the Standard Deviation of the portfolio returns?
A 7.13%
B 6.58%
C 8.25%
D 7%

14) The Portfolio consists of two securities, X and Y in the ratio of 70:30. Given that -
i) Standard Deviation of X is 10%
ii) Standard Deviation of Y is also 10% and covariance between them is 16%, what is the
portfolio risk?
A 13.77%
B 8.04%
C Nil
D 25%

Financial Planning Academy


15) Mohan invested Rs. 10,000 in XYZ mutual fund on 1/1/2000. He received cash
dividends of Rs. 200, Rs. 300, Rs. 200 & Rs. 300 on 31/12/2000, 31/12/2001, 31/12/2002
& 31/12/2003 respectively. He reinvested all the dividends received in the fund. He sold
the fund on 1/1/2004. What is the IRR if he sold the fund for Rs. 15,000?
A) 8.53%.
B) 8.83%.
C) 8.00%.
D) 10.67%.

Solution : Since Dividends are reinvested,


PV = -10000, N = 4, FV = 15000, IRR = 10.67%

16) A client purchased a zero coupon bond 6.5 years ago for Rs. 525. If the bond matures
today and the face value is Rs. 1,000, what is the average annual compound rate of return
(calculated semi-annually) that the client realised on her investments?
A) 10.40%
B) 5.08%
C) 11.34%
D) 10.16%

Financial Planning Academy

You might also like